You are on page 1of 53

Kansas State University

Department of Mathematics

Real and Complex Analysis


Qualifying Exams
(New System)
Solution Manual
Compiled by Peter Nguyen
Edited by R. B. Burckel

How to Use this Manual


This solution manual has been constructed so that each chapter represents a
specific semesters qualifying exam. Within each chapter, the problems are listed in
the order in which they originally appeared, and moreover, they have been transcribed
almost verbatim. The primary exception to this is the omission of hints.
Following the statement of each problem, the reader will find a list of key terms.
These are the important definitions and theorems relevant to the given solution. In
most cases, these are concepts or results that the test-taker is presumed to be knowledgable of. As such, these concepts have been used freely throughout the solutions.
After the list of key terms, the reader will find a solution to the given problem.
These solutions should not be preferred over others, although some are canonical.

iii

Notation
Z := Integers
N := {n Z : n 1}
N0 := N {0}
R := Real numbers
C := Complex numbers
S + := S (0, )

(S C)

D(z, r) := {w C : |z w| < r}
r) := {w C : |z w| r}
D(z,

(z C, r > 0)
(z C, r > 0)

D0 (z, r) := {w C : 0 < |z w| < r} (z C, r > 0)


D := D(0, 1)
:= D(0,
1)
D
D0 := D0 (0, 1)
C(z, r) := {w C : |z w| = r}

(z C, r > 0)

T := C(0, 1)
| | := Lebesgue measure on Rk

(k N)

E c := {x X : x 6 E}

1 x E
1E (x) :=

0 x E c
`
:= disjoint union

(E X)

B(x, r) := {y X : d(x, y) < r}

(d a metric on X, r > 0)

(E X)

iv

Contents
How to Use this Manual

iii

Notation

iv

Chapter 1. Spring 2004

Chapter 2. Fall 2004

Chapter 3. Spring 2005

18

Chapter 4. Fall 2005

27

Chapter 5. Spring 2006

38

Index

47

Bibliography

49

CHAPTER 1

Spring 2004
Problem 1.1. Let f : [0, 1] C be continuous and define F : := C\[0, 1] C
by

F (z) :=
0

f (t)
dt.
tz

Prove that F is holomorphic in .


Key terms: Moreras Theorem, Fubinis Theorem, Lebesgues Dominated Convergence Theorem.
Solution. Let a1 , a2 , a3 C be such that the closed convex hull of the oriented triangle T := [a1 , a2 , a3 , a1 ] is contained in . (Here we are using the notation
S
[a1 , ..., an ] := n1
m=1 [am , am+1 ], where [ai , aj ] is the oriented complex interval.) Then
by Fubinis Theorem, we have

Z 1 Z
1
f (t)
dt dz =
dz f (t)dt
F (z)dz =
0
T tz
T
0 tz
Z 1
Z 1
=
[2iIndT (t)] f (t)dt =
0 f (t)dt = 0.
Z Z

Z
T

As this holds for every triangle whose closed convex hull is contained in , Moreras
Theorem implies that F H().
Alternative Solution. This proof does not involve the use of Moreras Theorem.
First note that since f is continuous and [0, 1] is compact, we may find M N such
that |f (t)| M , for every t [0, 1]. Now, fix z0 . Since is open, we may find
r > 0 such that D := D(z0 , r) , and therefore, D [0, 1] = . In consequence, we
see that
(1.1.1)

|t z| r

(z D, t [0, 1]).

For z D, define gz : [0, 1] C by gz (t) := f (t)/[(t z)(t z0 )]. By (1.1.1), we


conclude that the collection each gz is well-defined, and moreover, |gz (t)| M/r2 ,
for every z D and t [0, 1]. From this, it follows that gz L1 ([0, 1]), for every
z D. Finally, as it is obvious that lim gz (t) = gz0 (t), for every t [0, 1], Lebesgues
zz0

Dominated Convergence Theorem ensures that


Z 1
Z 1
f (t)
f (t)
dt =
dt.
(1.1.2)
lim
2
zz0 0 (t z)(t z0 )
0 (t z0 )
For each z D0 (z0 , r), observe that
1
F (z) F (z0 )
=
z z0
z z0
(1.1.3)

1
z z0
Z 1

=
=

Z
0
1

Z
0

f (t)
dt
tz

1
0

f (t)
dt
t z0

f (t)(t z0 ) f (t)(t z)
dt
(t z)(t z0 )

f (t)
dt.
(t z)(t z0 )

Letting z z0 in (1.1.3) and appealing to (1.1.2) shows that F 0 (z0 ) exists, and in
fact,

f (t)
dt.
2
0 (t z0 )
As z0 is an arbitrary point of , we are finished.
0

F (z0 ) =

Problem 1.2. Let := {z C : 1 < |z| < 2}. Show that there does not exist a
sequence {Pn } of polynomials in z such that Pn (z) 1/z uniformly, for every z .
Key terms: uniform convergence, index of closed curve.
Solution. Assume, with a view to reach a contradiction, that {Pn } is a sequence
of polynomials converging to 1/z uniformly. Let : [0, 2] C be the closed curve
given by (t) = 23 eit . Since each polynomial is a continuous derivative in , we see
R
R
that Pn (z)dz = 0, for every n. On the other hand, (1/z)dz = 2iInd (0) =
R
2i. The uniform convergence of the Pn on to 1/z ensures that 2i = dz
=
z
Z
lim
Pn (z)dz = 0. This is the desired contradiction.
n

Problem 1.3. Evaluate


0

dx
.
1 + x7

Key terms: contour integration, pole, Residue Theorem, residue.


Solution. Let R > 1 and CR be the path given as the sum of the path1 t 7 Reit ,
for 0 t

2
7

and the oriented intervals [Re

Put f (z) :=

1
.
1+z 7

2i
7

, 0], and [0, R].

Note that f has exactly one simple pole lying inside the region
i

bounded by CR at z0 := e 7 . Since z07 = 1, we have


Z
(1.3.1)
f (z)dz = 2iRes(f, z0 ) = 2i lim (z z0 )f (z)
zz0

CR

= 2i lim

zz0

z z0
1
2i
= 2i d 7
= 6.
7
7
z z0
7z0
z z=z
dz
0

R
Now, since R > 1, we have 1+(Reit )7

R
,
R7 1

for all t, whence

Z 2
Z 2
7 iReit dt
7
Rdt
2R

=
.

0 1 + (Reit )7
R7 1
7(R7 1)
0
Thus,

R 2
0

iReit dt
1+(Reit )7

0, as R . Furthermore,

2
7

f (z)dz =

(1.3.2)
CR

0
2
7

=
0

Z R
z02 dr
dx
+
2 7
7
0 1 + (z0 r)
0 1+x
Z

R dx
iReit dt
2
+ 1 z0
.
7
1 + (Reit )7
0 1+x
iReit dt

1 + (Reit )7

Combining (1.3.1) with (1.3.2) and taking the limit as R gives


Z
dx
2i

.
= 6
=
2
7
1+x
7z0 (1 z0 )
7 sin 7
0
The final equality being obtained from the computation:
z06 z08 = e

6i
7

8i
7

= e 7 + e 7 = 2i sin(/7).

1See

[RUD, 217 - 218] for a brief discussion on what is meant by a sum of paths.

Problem 1.4. Let f : [0, ) C be a Lebesgue measurable function satisfying


|f (x)| aekx ,

()

for some a, k (0, ). Put := {z C : Im z > k}.


(a) Show that for every z , the function gz : [0, ) C defined by gz (t) :=
eitz f (t) is in L1 ([0, )).
(b) Prove that the map F : C defined by F (z) :=

R
0

gz (t)dt is holomorphic

in .
Key terms: Lebesgue measure, Lebesgues Dominated Convergence Theorem, complex differentiability, (Lebesgue) measurable function.
Solution. (a) Clearly, each gz is measurable, being the product of measurable functions. Now, fix z := x + iy and observe
Z
Z
Z
it(x+iy)

|gz (t)| dt =
e
f (t) dt
ety aekt dt
0
0
0
Z
a
= a
et(yk) dt =
< ,
yk
0
since y = Im z > k. This completes the proof of (a).
(b) Fix z0 = x0 + iy0 . Choose r > 0 such that D := D(z0 , r) . Let
R
0 , r/2) with z 6= z0 . Since it
t [0, ) and z D(z
eit d = eitz eitz0 , we have
[z0 ,z]

itz

Z
Z

e eitz0
it
it
itz
it
itz
0
0

ite
e
d

e
d
=

z z0
z z0
z z0 [z0 ,z]
[z0 ,z]

Z
it
it

=
e eitz0 d
z z0 [z0 ,z]

length[z0 , z] sup eit eitz0


|z z0 |
[z0 ,z]
it itz0
(1.4.1)
t sup e + e
[z0 ,z]


2t sup eit
[z0 ,z]

= 2t sup etIm
[z0 ,z]

= 2tetk0 ,

0 , r/2) is a compact subset of .2 By the hypothesis


for some k0 > k, since [z0 , z] D(z
(), we find that
tk

te 0 f (t) ate(k0 k)t L1 ([0, )).

(1.4.2)

Combining (1.4.1) and (1.4.2) with Lebesgues Dominated Convergence Theorem, we


find that

Z
F (z) F (z0 )
itz0

ite f (t)dt
lim
zz0
z z0
0

Z itz
e eitz0
itz0
= lim
ite
f (t)dt
zz0 0
z z0

Z
eitz eitz0
itz0
ite
f (t)dt
=
lim
zz0
z z0
0

= 0.
Thus, F 0 (z0 ) exists and, in fact, F 0 (z0 ) =

R
0

iteitz0 f (t)dt.

Problem 1.5. A Lebesgue measurable subset of R is said to be negligible if it has


Lebesgue measure zero. Prove that a subset A of R is negligible if and only if there
T
exists a sequence {Un } of open subsets of R such that lim |Un | = 0 and A n Un .
n

Key terms: Lebesgue measure, Lebesgue measurable set, negligible set, complete
measure space.
Solution. () As A has Lebesgue measure zero, it follows from the very definition
of Lebesgue (outer) measure that, for every n N, we may find a sequence {In,m }
m=1
P
S
S
1
of open intervals such that A m In,m and m |Im,n | < n . Put Un := m In,m .
Then Un is open, being the union of open sets. Since A Un , for each n, it must be
T
P
that A n Un . Now, by countable subadditivity, |Un | m |In,m | < n1 , for each
n N, whence |Un | 0, as n .
() Let {Un } be the sequence of open sets given by the hypothesis of this impliT
cation. Observe that 0 | m Um | |Un |, for every n. Since |Un | 0, as n (by
2This

is a standard way to circumvent the Mean-Value Theorem, which is unavailable for

complex-valued functions.

assumption), it must be that |

T
m

Um | = 0. Now,

T
m

Um is Lebesgue measurable, be-

ing the countable intersection of open (and therefore Borel) sets. Thus, A is Lebesgue
measurable, since it is the subset of a set of measure zero and since Lebesgue measure
is complete. Finally, A has measure zero, by monotonicity.

Problem 1.6. Let 1 q < p .


(a) Prove that Lp ([0, 1]) Lq ([0, 1]).
(b) Show (by example) that the inclusion in (a) is strict.
(c) Give an example of a measure space (X, A , ) for which the inclusion Lp (X)
Lq (X) holds.
Key terms: Lebesgue measure, Lp -space, counting measure.
Solution. (a) See the solution for Problem 5.1(a).
(b) Let us first handle the case when p = . Consider the function f (x) :=
x

1
2q

Lq ([0, 1]). For M > 0, let bM := min {M 2q , 1}. It is easy to check that
{x (0, 1) : |f (x)| > M } = (0, bM ),

from which it follows that ess sup f = , since |(0, bM )| = bM > 0. Therefore,
f 6 L ([0, 1]). So, the inclusion of (a) is strict in this case.

1 1
Now, fix 1 q < p < and let r p , q . Notice that 1 qr > 0, while
1 pr < 0. So,
Z 1 q
Z 1

1
1
1
1
qr
1qr 1
1qr
dx = lim
x
dx
=
lim
x
=
lim
(1t
)
=
,
xr
t
t0+ t
t0+ 1 qr
t0+ 1 qr
1 qr
0
while
Z
Z 1 p
1
dx = lim
xr
t0+
0

1
t

xpr dx = lim+
t0

1
1
1
x1pr t = lim+
(t1pr 1) = .
t0 pr 1
1 pr

Thus, 1/xr Lq ([0, 1])\Lp ([0, 1]), showing the inclusion is strict, in this case as well.
(c) One can construct a multitude of trivial examples easily: merely take X :=
any set, A := any -algebra on X (one may pick the power set of X for concreteness),
and finally, := the zero measure on X (i.e., (A) = 0, for all A A ).

Of course, a nontrivial example is to be sought out. Perhaps, the simplest such


example is to take X := N, A := the power set of N, and := counting measure.
P
q
Here if f Lq (X), then by definition
n=1 |f (n)| < . Consequently, we may
find N N such that |f (n)|q 1, for each n N . So, for n N , we have
|f (n)|p |f (n)|q , whence

|f (n)|p

N
1
X

n=1

|f (n)|p +

n=1

|f (n)|q < .

n=N

This shows that f Lp (X), and therefore, Lp (X) Lq (X).

p
Problem 1.7. Let p [1, ) and suppose {fn }
n=1 L (R) is a sequence that

converges to 0 in the p norm. Prove that we may find a subsequence {fnk } such that
fnk 0 a.e.
Key terms: pointwise convergence, p-norm convergence, almost everywhere, complete metric space, Beppo-Levi Theorem.
Solution. Choose n1 N such that kfn kp < 12 , for all n n1 . Having found this n1 ,
we may find n2 N such that n2 > n1 and kfn kp <

1
,
22

for all n n2 . Continuing in

this manner, we may inductively find nk N such that nk > nk1 and kfn kp <
whenever n nk . Set fn0 = 0. Note that kfnk kp <

1
,
2k

1
,
2k

for all k 1.

By the Beppo-Levi Theorem (or Lebesgues Monotone Convergence Theorem), we


have

Z "X

Consequently,

#
|fnk (x)|

dx =

k=1

kfnk kpp

k=1

k=1

2k

2p

1
< .
1

k=1

|fnk (x)| is finite for a.e x R, and therefore, convergent for

a.e. x R. For these x, it follows from the convergence of the above series that
|fnk (x)|p 0, as k , or equivalently, that fnk (x) 0, as k .

Problem 1.8. Let f be an entire function having the property that


f (z + m + ni) = f (z)

(z C, m, n Z).

Prove that f is constant.


Key terms: entire function, Liouvilles Theoerem.
Solution. Put S := [0, 1] [0, 1]. The periodicity condition on f gives that f (C) =
f (S). Since S is compact and f is continuous (it is holomorphic), it follows that f is
bounded on S, and therefore, f is bounded on C. By Liouvilles Theorem, we deduce
that f is constant.

CHAPTER 2

Fall 2004
Problem 2.1. Compute
Z
(a)
e[x] dx, where [x] := max {n Z : n x}, the integer part of x.
0

Z /2
cos x x R\Q
(b)
f (x)dx, where f (x) :=
.

0
sin x x Q
Key terms: Lebesgues Monotone Convergence Theorem, Beppo-Levi Theorem.
Solution. (a) Applying the Beppo-Levi Theorem (or Lebesgues Monotone Convergence Theorem) and the fact that [x] = n, on [n, n + 1), for n N0 , we get
Z

[x]

[x]

dx =

"
X

1[n,n+1) (x) dx =

n=0

Z
X
n=0

Z
X
n=0

1[n,n+1) (x)dx =

e[x] 1[n,n+1) (x)dx

en = e/(e 1).

n=0

(b) Put I := [0, /2]. Since integrals over sets of measure zero are 0, we find that
Z

/2

f (x)dx =
0

+
Z

I\Q

f (x)dx =
IQ

cos xdx +
I\Q

cos xdx =
Z

Z
Z

cos xdx +

I\Q

I\Q

sin xdx
IQ

cos xdx
IQ

/2

cos xdx = 1.
0

Note that the above functions are measurable, since in a complete measure space we
may arbitrarily redefine functions over sets of measure zero and retain measurability.

Problem 2.2.

(a) Does the function

x sin 1 0 < x 1
x
f (x) :=

0
x=0

have bounded variation?


(b) Compute Var[0,50] (ex ), the total variation of ex on the interval [0, 50].
Key terms: bounded variation, total variation.
Solution. (a) No. For each integer N 3, let PN denote the partition 0 := x0 < x1 <
... < xN := 1, where xn :=

+(N n)
2

, for n = 1, ..., N 1. Since sin x1n = (1)N n ,

for each n = 1, ..., N 1, the total (or absolute) variation TN corresponding to this
partition satisfies
TN

N
1
X

1
(1)N n
+(N
n)

n=2

(2.2.1)

N
1
X
n=2

N
1
X

+(N n)
2

2
(N n+2)

n=2

+[N (n1)]
2

+[N (n1)]
2
N
X

N
1
X

(1)N n+1

1
N n+1

1
N n+2

n=2

2
.
n

n=3

Now, the total variation T satisfies T TN , for all N N. Hence, letting N in


(2.2.1) verifies that f is not of bounded variation.
(b) Let PN := 0 = x0 < x1 < ... < xN = 50 be a partition of [0, 50]. By the

Mean-Value Theorem, we may find xn [xn1 , xn ] such that exn =


N
X
n=1

|exn exn1 | =

N
X

exn exn1
.
xn xn1

Hence,

exn (xn xn1 ).

n=1

Letting our partitions become finer and finer, one version of the Riemannian theory
R 50
of integration ensures the sum on the right converges to 0 ex dx = e50 1. On the
other hand, the left sum converges to the total variation.
More generally, the above argument shows that one has Var[a,b] f =

Rb
a

|f 0 (x)| dx,

for every continuously differentiable function f on a compact interval [a, b].

10

Problem 2.3. Suppose f H(\ {0}) and that 0 is either a pole of order m for
f or a removable singularity whose removal results in 0 being a zero of order m for
f . Show that 0 is a first order pole of f 0 /f having residue either m or m.
Key terms: pole, removable singularity, residue
Solution. In either case, we may write f (z) = z n g(z), where n := m, g H()
and g(0) 6= 0. Since g(0) 6= 0 and g is continuous there (it is differentiable), we may
find r > 0 such that g is zero-free on D := D(0, r). Thus,
(2.3.1)

f 0 (z)
z n g 0 (z) + nz n1 g(z)
g 0 (z)
=
=
+ nz 1
n
f (z)
z g(z)
g(z)

(z D\ {0}).

As g H(D) and g is zero-free on D, it follows that g 0 /g H(D), whence (2.3.1)


ensures that f 0 /f has a simple pole at 0 with residue n = m.

Problem 2.4. Show that a nonconstant entire function maps the plane onto a
dense subset of the plane.
Key terms: Casorati-Weierstrass Theorem, entire function
Solution. The proof to follow mimics the canonical proof of the Casorati-Weierstrass
Theorem. Let f be a nonconstant entire function. And suppose, in order to reach a
contradiction, that f (C) is not dense in C. Then we may find z0 C and r > 0 such
that f (C) D(z0 , r) = . Consequently,
(2.4.1)

|f (z) z0 | r,

for every z C. This implies that the function g : C C defined by g :=


is entire. Now, (2.4.1) also implies that |g|

1
r

1
f z0

< , and so, Liouvilles Theorem

ensures that g is constant. This, in turn, forces f to be constant. (Specifically,


f=

1
c

+ z0 , where g = c.) This contradiction implies f (C) is dense in the plane.

H() is
Problem 2.5. Let be a bounded region. Suppose that f C()
zero-free and |f | = C on .

11

(a) Prove that f must be constant.


(b) Is the boundedness condition imposed on essential?
Key terms: Maximum Modulus Principle.
Solution. (a) First note that since f is zero-free, it must be that C > 0. By the
Maximum Modulus Principle,
(2.5.1)

|f (z)| kf k := max {|f (z)| : z } = C

(z ).

H(). Consequently, the


Now, since f is zero-free, we evidently have 1/f C()
Maximum Modulus Principle applies to 1/f , and therefore,
(2.5.2)

|1/f (z)| k1/f k = 1/C

(z ).

Inequalities (2.5.1) and (2.5.2) combine to show that |f (z)| = C on . Thus, |f |


attains its maximum at every point of , and so, the Maximum Modulus Principle
forces f to be constant on .
(b) Yes. Consider the region := {z C : 0 < Re z}. The exponential map ez is
zero-free and holomorphic on , as well as, continuous and identically equal to 1 in
modulus on the boundary of . Yet, this map fails to be constant on . In fact, the
continuity of f shows that this constant must be C.

Problem 2.6. Does there exists a Lebesgue measurable subset E of R such that
|EI|
|I|

= 12 , for every interval I R.

Key terms: Lebesgue measure, Lebesgue point, almost everywhere


Solution. No. To verify this, we will need the following general results applied to
the one-dimensional scenario:
Lemma 2.6.1. Let E be Lebesgue measurable subset of Rk . Then for a.e. x Rk ,
the limit
|E B(x, r)|
r0
|B(x, r)|
exists and is equal to 1 for a.e. point of E and 0 for a.e. point of E c .
lim

12

Proof. Assume for a moment that the lemma holds for measurable sets having
finite measure. For each n N, put En := E B(0, n). Each En has finite measure,
being a subset of a set of finite measure, namely B(0, n). Thus, the lemma applies to
|En B(x, r)|
each En . For each n, let An be the set of points x En such that lim
r0
|B(x, r)|
exists and is equal to 1. Likewise, let Bn be the set of points x Rk \En for which
the limit exists equal to 0. Since the lemma is assumed to hold for finite measured

T
sets, we have that Rk \An = 0 and Rk \Bn = 0, for each n. Put A :=
n=1 An and
k k
T
B := n=1 Bn and notice that R \A = R \B = 0, being the countable union of
null sets.
Fix x A. Since En B(x, r) En+1 B(x, r), for all n, and since

S
n=1

En

B(x, r) = E B(x, r), we conclude that


(2.6.1)

|E B(x, r)|
|En B(x, r)|
= lim
.
n
|B(x, r)|
|B(x, r)|

Let > 0 be given. By (2.6.1), we may find N 1 so large so that


(2.6.2)

|E B(x, r)| |EN B(x, r)|


< /2.

|B(x, r)|
|B(x, r)|

Notice that the quantity on the left is well-defined, since 0 < |B(x, r)| < . As

|EN B(x,r)|
x A AN , we may find r0 > 0 such that 0 < r < r0 implies |B(x,r)| 1 < /2.

1
This fact along with (2.6.2) and the triangle inequality shows that |EB(x,r)|
< ,
|B(x,r)|
|E B(x, r)|
whenever 0 < r < r0 . Hence, lim
exists and is equal to 1 for every x A.
r0
|B(x, r)|
|E B(x, r)|
In a similar (in fact, easier) fashion, we see that lim
exists and is equal
r0
|B(x, r)|
to 0 for every x B. Let F and G be for E and Rk \E as An and Bn are for En
and Rk \En , respectively. Then we have just proved that A F and B G so that

Rk \F Ac and Rk \G B c , and therefore, Rk \F = Rk \G = 0.


The above argument guarantees that it suffices to prove the lemma with the extra
condition that |E| < . In this case, put
1
vr (x) :=
|B(x, r)|

Z
|1E (y) 1E (x)| dy
B(x,r)

13

for 0 < r and x Rk . Since E has finite measure, 1E L1 (Rk ), whence almost every
point of Rk is a Lebesgue point of 1E ; i.e., lim vr (x) = 0, for a.e. x Rk .
r0

Case 1: x E. Then |1E (y) 1E (x)| = 1E c (y), and therefore,


vr (x) =

|E c B(x, r)|
|B(x, r)| |E B(x, r)|
|E B(x, r)|
=
=1
|B(x, r)|
|B(x, r)|
|B(x, r)|

(r > 0).

Case 2: x E c . Then we directly have


vr (x) =

|E B(x, r)|
|B(x, r)|

(r > 0).

Since lim vr (x) exits and is equal 0 for a.e. x Rk , the proof is complete.
r0

Proposition 2.6.2. Let E Rk be Lebesgue measurable and let > 0. Then


there exists x Rk and r > 0 such that either >

|EB(x,r)|
|B(x,r)|

or 1 <

|EB(x,r)|
.
|B(x,r)|

Proof. Since Rk is the disjoint union of E and E c , either |E| > 0 or |E c | > 0.
In the former case, the preceding lemma implies the existence of an x E such that
|E B(x, r)|
= 1. Thus, we may find some r > 0 such that 1 < |EB(x,r)|
. So,
lim
|B(x,r)|
r0
|B(x, r)|
we are finished with this situation.
If, however, |E c | > 0, then applying the preceding lemma again shows that there
|E B(x, r)|
= 0. So, we may find some r > 0 such
exists an x E c such that lim
r0
|B(x, r)|
that > |EB(x,r)|
. This finishes the proof.
|B(x,r)|

Returning to Problem 2.6, the answer is no. To see this, we simply take k = 1
and =

1
2

in the preceding proposition.

2
x y2

2
2 2
Problem 2.7. Let f (x, y) := (x + y )

(x, y) 6= (0, 0)

(x, y) = (0, 0)

(a) Show that


Z

dx
0

f (x, y)dy 6=
0

dy
0

14

f (x, y)dx.
0

(b) To save Fubinis Theorem, use polar coordinates to verify that

|f (x, y)| dxdy = .


0

Key terms: polar coordinates, Fubinis Theorem


Solution. (a) First, some clarification is due regarding the meaning of the iterated
integrals. The quantity
Z

Z
dx
A

f (x, y)dy,
B

is the Lebesgue integral of the function given by F (x) :=

R
B

f (x, y)dy, provided this

function is well-defined and Lebesgue integrable. Similarly, for the other iterated
integrals in (a). In our present situation,

F (x) :=
0

y
x2 + y 2

dy =

y y=1
1
,
=
x2 + y 2 y=0 x2 + 1

so that the measurability of F is evident. (F is, in fact, continuous on (0, 1].) Thus,

1
0

1
F (x)dx = arctan x0 = /4.

On the other hand, since f (x, y) = f (y, x), we see that the remaining iterated
integral exists and evaluates to /4.

15

(b) Note that the circular sector S = rei : 0 r 1, 0 /2 is contained


in the rectangle R = {(x, y) : 0 x, y 1}. Thus,
Z

Z Z

|f (x, y)| dxdy =


0

|f (x, y)| dxdy


Z Z

|f (x, y)| dxdy


S

"Z

/2

|f (r cos , r sin )| d rdr


0

cos 2

=
r2 d rdr
0
0
!
Z 1 Z /4 Z /2
1
=

cos 2d
dr
r
0
0
/4
Z 1
1
=
dr
0 r
Z

/2

= .
Problem 2.8. Prove that f 0, whenever f H(C) L1 (R2 ).
Key terms: Cauchys Formula, polar coordinates, Lp -space.
Solution. For r > 0 and z C, let r,z be the parameterization of the boundary of
the circle centered at z with radius r given by r,z (t) := z + reit , 0 t 2. Since f
is entire and C is convex, we may apply Cauchys formula to find that
Z
Z 2
1
f ()
1
f (z + reit )
f (z) =
d =
(rieit dt)
2i r,z z
2i 0 (z + reit ) z
Z 2
1
=
f (z + reit )dt,
2 0
and so,

(2.8.1)

1
|f (z)|
2

f (z + reit ) dt.

16

Integrating both ends of (2.8.1) with respect to rdr over [0, R] and appealing to
Fubinis Theorem gives

(2.8.2)

Z R Z 2

1
R2
it
f (z + re ) dt rdr
|f (z)|
2
2 0
0
Z
1
=
|f (z + w)| d2 (w)
2 E
Z
1
1

kf k1 ,
|f | =
2 R2
2

where E := {(x, y) D(0, r) : 0 < x, y} and 2 is Lebesgue measure in R2 . Since


(2.8.2) is clearly equivalent to |f (z)|

kf k1
,
R2

which holds for all z C and R > 0, we

may deduce that |f (z)| = 0, for all z. Equivalently, f 0.

17

CHAPTER 3

Spring 2005
Problem 3.1. Compute
Z 2
dt
(a)
.
cos t 2
0
(b) Res(z n e10/z , ), n N.
(c) Res (exp(z 2 )/z 2n+1 , 0), n N.
Key terms: residue, contour integration, Residue Theorem, pole.

Solution. (a) Let be the curve defined by t 7 eit , for 0 t 2. If we write


z = (t), then cos t = (z + z 1 )/2, dt = idz/z, and so,
Z

(3.1.1)
0

dt
=
cos t 2

(idz/z)
= 2i
(z + z 1 )/2 2

z2

dz
.
4z + 1

Let f (z) be the function in the integrand of (3.1.1). The quadratic formula shows

that f has simple poles at z0 = 2 2 3 and z1 = 2 + 2 3. Among these, z0 lies


in the region bounded by the image of , while z1 lies outside this region. Thus, the
integral on the far right of (3.1.1) is equal to

3i
2i
1
=
=
.
2iRes(f, z0 ) = 2i lim (z z0 )f (z) = 2i lim
zz0
zz0 z z1
z0 z1
6
Substitution of this into (3.1.1) yields
Z

2
0

dt
3
=
.
cos t 2
3

(b) Recall that Res(f, ) := Res(f (1/z), 0). Now,


n 10/(1/z)

(1/z) e

=e

10z

/z = (1/z )

X
k=0

(10z) /k! =

(10k /k!)z kn .

k=0

Hence, the coefficient of the z 1 term is Res(z n e10/z , ) = 10n1 /(n 1)!.

18

(c) Observe that


2

exp(z )/z

2n+1

= (1/z

2n+1

X
X
2 k
)
(z ) /k! =
z 2k2n1 /k!.
k=0

k=0

So, the coefficient of the z 1 term is Res(exp(z 2 )/z 2n+1 , 0) = 1/n!.

Problem 3.2. Let {Ej }m


j=1 be a collection of measurable subsets of [0, 1]. Assume
that this collection has the property that every x [0, 1] belongs to at least n of the
Ej , where n m. Prove that |Ej |

n
,
m

for some j.

Key terms: Lebesgue measure.


Solution. Since every x [0, 1] belongs to at least n of the Ej , it must be that
Pm
j=1 1Ej (x) n, for every x [0, 1]. Now, suppose, in order to reach a contradiction,
that |Ej | <

n
,
m

for every j. (As all of our integrals will be over [0, 1] we will suppress

the integrating set.)


Z
n = n |[0, 1]| =

Z X
m

1Ej =

m Z
X

j=1

j=1

1Ej

m
X

m
X
n
= n.
=
|Ej | <
m
j=1
j=1

The appearance of the strict inequality is the sought contradiction, and therefore, the
desired conclusion follows.

Problem 3.3. Given 1 p < q < r < , prove that


Lp (X, ) Lr (X, ) Lq (X).
Key terms: Lp -space.
Solution. Let f Lp (X) Lr (X). Set E := {x : |f (x)| > 1} and notice that
|f |p < |f |q < |f |r on E, while |f |r |f |q |f |p on E c . Therefore,
Z
Z
Z
Z
Z
Z
Z
q
q
q
r
p
r
|f | =
|f | +
|f |
|f | +
|f |
|f | +
|f |p < .
X

Ec

Ec

19

Problem 3.4. Let f L1 (X, ). Prove that for every > 0, there exists > 0
such that

f d < ,

whenever A is a measurable subset of X with (A) < .


Key terms: Lp -space, Lebesgues Monotone Convergence Theorem.
Solution. Let > 0 be given. We will actually prove the existence of > 0 such
that

Z
|f | d < ,
A

for every measurable A X with (A) < . Once this is shown, we will be finished
R
R
because the inequality A f d A |f | d holds. Since we are only concerned with
|f |, we may assume that f = |f |; that is, we assume f is nonnegative.
For n N, define fn := min {f, n}. Notice that 0 fn (x) fn+1 (x) f (x) and
fn (x) f (x) holds for every x X and n N. So, by Lebesgues Monotone ConverR
R
R
gence Theorem, we have lim X fn d = X f d, or equivalently, X (f fn )d 0,
n
R
as n . Hence, we may choose N so large so that X (f fN )d < 2 . Having
chosen this N , we now choose > 0 such that N 2 . So, for measurable A X
with (A) < , we get
Z
Z
Z
Z
Z
f d =
(f fN )d +
fN d
(f fN )d +
N d
A

<
+ N (A) < + N < .
2
2

Problem 3.5. Is there an f H(D) such that mn (f ) , as n , where

mn (f ) = inf |f (z)| : 1 n1 < z < 1 ?


Key terms: zero-set, Bolzano-Weierstrass, Maximum Modulus Principle.
Solution. No. To see this, assume the contrary. That is, suppose there exists
f H(D) such that mn (f ) , as n . Choose n so large so that mn (f ) > 0.
We conclude from the definition of mn (f ) that f is zero-free on the annulus A :=

20


z :1

< z < 1 . Thus, all the zeros of the nonconstant function f must lie in

0, 1 1 . Since the zero-set of f contains no limit point of D, we


the compact set D
1
n

conclude (by Bolzano-Weierstrass) that f has only finitely many zeros, say {z1 , ..., zk }.
By iterated use of Theorem 10.18 in [RUD], we may write
f (z) = (z z1 )m1 (z zk )mk g(z),
where g H(D) and g has no zero in D. Observe that
|f (z)| = |(z z1 )m1 (z zk )mk g(z)| = |z z1 |m1 |z zk |mk |g(z)|
2m1 ++mk |g(z)| ,
for every z D. From this it follows that mn (g) , as n . Since


g 1 1 ei : 0 2 |g(z)| : 1 1 < z < 1 ,

2n
n
for each n, it must be that

min g 1
ei mn (g),
02
2n

for each n. By the corollary to Theorem 10.24 in [RUD] (i.e. the Minimum Modulus
Principle, which is just the Maximum Modulus Principle applied to 1/g, g being
zero-free), we may deduce that |g(0)| mn (g), for each n, which is impossible if
mn (g) . This is the desired contradiction.

Problem 3.6. Let f : C, where is open.


(a) True or false: (i) If f is holomorphic and ef is constant, then f is constant.
(ii) Does the answer change if f is only assumed continuous?
(b) Is a bounded function that is harmonic on all of C constant?
Key terms: harmonic function, entire function, Liouvilles Theorem
Solution. Recall that we are assuming to be a region, and so, it is connected.
(a) It suffices to show the following: If f is continuous and ef is constant, then f is
constant, since holomorphic functions are continuous. The non-zero constant function
ef is ew , for some w C. Hence, ef (z)w = 1, or equivalently, f (z) w 2iZ, for

21

all z . As f (z) w is a continuous mapping of the connected region into the


discrete space 2iZ, it must be that f w = 2in, for some fixed integer n, whence
f (z) = w + 2n. That is, f is constant.
(b) As the real and imaginary parts of f satisfy the same conditions as f does, we
may without loss of generality assume that f is real-valued. As such, on each closed
N ), N N, f is the real part of a holomorphic function that is defined
disk D(0,
uniquely up to a pure imaginary additive constant (see [RUD, 235 - 236]). Adjusting
these constants at each radius, we may conclude that f is the real part of an entire
function, say F = f + iv. Now, since the exponential function is nonnegative and
strictly increasing on the real axis, we find that
F f +iv f iv f
e = e
= e e = e = ef e|f | eM ,
where M is a (finite) bound for f . Consequently, we may apply Liouvilles Theorem
to the entire function eF to conclude that eF is constant. From part (a), F must be
constant, and therefore, f is constant.

Problem 3.7. Let I be a nonempty (compact) interval in R and let f : I C


be real-analytic, by which it is meant that in a neighborhood of every point of I, f is
represented by a convergent power series. Show that we may extend f to a holomorphic
function on some open subset of C that contains I. That is, prove that there exists
open C and F H() such that I and F |I f .
Key terms: power series, Uniqueness Theorem for Holomorphic Functions, uniform
convergence.
Solution. It turns out that the assumption that I be compact is not essential. So, we
P
n
will dispense with it. Now, for each p I, let
n=0 cn (p)(x p) be the convergent
power series representation of f about p, given by the hypothesis, and let Rp denote
P
n
the radius of convergence of this power series. Define fp (z) :=
n=0 cn (p)(z p) ,
for z D(p, Rp ). By assumption, Rp > 0. Choose rp such that 0 < rp < Rp . By

22

elementary power series theory, we have that the series for fp converges uniformly and
absolutely on Dp := D(p, rp ). Note that fp (x) = f (x), for every x I [p rp , p + rp ].
S
Put := pI Dp and define F : C by F (z) := fp (z), whenever p I is such
that z Dp . Provided that F is well-defined, it is obvious that F H(), since F is
representable by a power series about every point. Furthermore, that F |I f is also
self-evident, by construction. Hence, we seek to show that F is well-defined.
Suppose p, q I are such that z Dp Dq . Without loss of generality, we may
assume that p < q. Note that the triangle inequality ensures that q p < rq + rp . So,
if there were an x with p < x < q and x 6 Dp Dq , then we would have
q p = (q x) + (x p) rq + rp > q p.
This contradiction forces (p, q) Dp Dq . Hence, if (p, q) Dp Dq = , then
{Dp , Dq } would constitute a separation of the connected interval (p, q), see [RUD,
196]. From this impossibility, we may conclude that there exists x (p, q) Dp Dq .
It follows that we may find > 0 so small so that (x , x + ) Dp Dq . Since the
interval (x , x + ) lies inside I, we see that fp and fq agree on (x , x + ). As this
interval obviously has a limit point in the region Dp Dq , the Uniqueness Theorem
for Holomorphic functions guarantees that fp and fq agree on all of Dp Dq , whence
F is well-defined.

Problem 3.8. Let {rk }


k=1 be one-to-one enumeration of the rational numbers in
(0, 1]. For each k N, let pk , qk N be relatively prime positive integers such that
rk = pk /qk . Define fk : [0, 1] R by fk (x) := exp[(pk xqk )2 ]. Prove that fk 0
in measure, as k , yet, for each x [0, 1], the pointwise limit of the the sequence
{fk (x)}
k=1 does not exist.
Key terms: convergence in measure, pointwise convergence
Solution. Let us prove some helpful lemmas:
Lemma 3.8.1. Suppose a, b, c, d N are such that gcd(a, b) = gcd(c, d) = 1 and
a/b = c/d. Then a = c and b = d.

23

Proof. Since gcd(a, b) = 1, we may find m, n Z such that am + bn = 1. Now,


combined with the (assumed) fact that a/b = c/d, we get
c = c(am + bn) = acm + bcn = acm + adn = a(cm + dn).
Thus, a divides c. Symmetrically, we find that c divides a, whence a = c. Analogously,
we get that b = d, completing the proof of the lemma.

Corollary 3.8.2. Define : Q+ N N by the condition that (r) :=


(1 (r), 2 (r)) if and only if r = 1 (r)/2 (r) and gcd(1 (r), 2 (r)) = 1. Then is
a well-defined injective function.
Proof. For r, s Q+ , the following are equivalent:
(1) r = s.

(2) 1 (r)/2 (r) = 1 (s)/2 (s).

(3) 1 (r) = 1 (s) and 2 (r) = 2 (s). (4) (1 (r), 2 (r)) = (1 (r), 2 (r)).
(5) (r) = (s).
Hence, is well-defined and injective.

Lemma 3.8.3. For each N N, put SN := {r Q (0, 1] : 2 (r) = N }. Then


N

SN = Q (0, 1] and |SN | N , for each N .

Proof. That

`
N

SN is all of Q (0, 1] is obvious, since every positive rational

r has a natural number as its denominator 2 (r). That the SN are disjoint follows
just as easily, since 2 is a function and so must assign only one value for any given
input. To see that the cardinality of each SN is at most N , suppose r SN . In order
that r 1, we must have 1 (r) 2 (r) = N . There are precisely N natural numbers
having this property, so that there are at most N possible numerators for r. Hence,
there are at most N possible values for r. That is, there at most N elements in SN .

Corollary 3.8.4. lim qk = .


k

24

Proof. It is a matter of definition (and the uniqueness element of Lemma 3.8.1)


that qk = 2 (rk ). For M N, put TM := {r Q (0, 1] : 2 (r) M }. Then TM =
`M
N =1 SN , where the SN are as above. Since |SN | N and TM is the disjoint union,
P
we get |TM | M
N =1 N = M (M + 1)/2. In particular, TM has finite cardinality for
each M . So, for each M N, the fact that k 7 rk is injective guarantees that we
may set K := max {k N : rk TM }. Thus, for each k > K, rk 6 TM , and therefore,
qk > M . This proves the corollary.

Lemma 3.8.5. Given x (0, 1] and prime n, there exists ln such that |x rln | <
1/qln .
Proof. Put Ij :=

j1
n

`
, nj , for j = 1, ..., n. Then (0, 1] = nj=1 Ij . So, x Ij , for

some j, and therefore |x j/n| < 1/n. By hypothesis, the rational number j/n is rln
for a unique ln N. Since n is prime gcd(j, n) = 1, and so, if rln = j/n, then pln = j
and qln = n. Hence, |x rln | = |x j/n| < 1/n = 1/qln .

We are finally prepared to tackle the proof of Problem 3.8. Notice that we must
formally verify that each fk is well-defined, but this is handled by Corollary 3.8.2.
Now, fix x (0, 1]. Choose so that 0 < < x. Since there are infinitely many
rational numbers in (0, ), we may find a subsequence {rkm } (0, ). Notice that
0 < |x | < |x rkm |, for each m. Multiplying through by qkm , squaring, and then
taking exponentials gives fkm (x) < exp[qkm (x )2 ]. Letting m and appealing
to Corollary 3.8.4 shows that fkm (x) 0.
On the other hand, Lemma 3.8.5 yields a (different) subsequence {rln } such that
|x rln | < 1/qln , for each n. In consequence, 1 < (pln xqln )2 so that fln (x) > 1/e.
It follows that the pointwise limit fk (x) cannot exist. And this is true of every
x (0, 1].
It remains to be seen that the fk converge in measure to the zero function. To this
end, let 0 < < 1 be given. Observe that if x (0, 1] is such that exp[(pk xqk )2 ] =
p
|fk (x)| > , then |x rk | < (1/qk ) ln(1/). Thus, {x : fk (x) > } (rk k , rk +k ),

25

p
where k := (1/qk ) ln(1/). Hence, |{x : |fk (x)| > }| |(rk k , rk + k )| = 2k .
By Corollary 3.8.4, k 0 as k , whence we may choose K so large so that
2k < , for every k K. And so, fk 0 in measure.

Problem 3.9. Let (X, A , ) be a finite measure space and f a nonnegative mea
X
1
surable function on X. Then f L (X) if and only if
2n (Sn ) < , where
n=0

Sn := {x X : f (x) 2n }.
Key terms: finite measure space, Lp -space.

Solution. () For n N0 , let Tn := {x X : 2n f (x) < 2n+1 }. Also, put A :=


`
P
{x : 0 f (x) < 1}. Note that X = A q ( n Tn ) so that 1X = 1A +
n=0 1Tn . Also,
notice that 1Tn 1Sn , since Tn Sn . Thus,

X
X
X
f = f 1A +
1Tn = f 1A +
f 1Tn 1A +
2n+1 1Tn
n=0

1X + 2

n=0

n=0

2n 1Sn L1 (X),

n=0

2n (Sn ) are both assumed finite.


`
() Let Tm be as above and
notice that Sn =
m=n Tm , for each n N0 . Define

0 m < n
a : N0 N0 {0, 1} by an,m :=
. Then

1 m n
"
#

X
X
X
X
X
n
n
n
2 1Sn =
2
1Tm =
2 1Tm an,m =
2n 1Tm an,m

since (X) and

n=0

n=0

n=0

m=n

m1

X
X
m=0

= f

n=0

n=0 m=0

1Tm

1Tm =
!

(2 1) 1Tm

m=0

m=0 n=0

X
m=0

2 1Tm

f 1Tm

m=0

f 1X = f L1 (X).

m=0

26

CHAPTER 4

Fall 2005
Problem 4.1. Complete the following:
(a) State and prove the Schwarz lemma for holomorphic self-maps of D.
(b) From (a), conclude that any conformal automorphism of D that fixes zero
must be a rotation.
Key terms: Schwarz Lemma, conformal mapping, rotation, removable singularity,
Maximum Modulus Principle, Chain Rule.
Solution. (a) Here essentially is the version found in [RUD, 254] and its proof:
Theorem 4.1.1 (Schwarz Lemma). Suppose f H(D) is such that kf k 1
and zero is fixed by f . Then
(4.1.1)

|f (z)| |z|

(z D)

and
(4.1.2)

|f 0 (0)| 1.

If equality holds in (4.1.1) for some z D0 or if equality holds in (4.1.2), then there
exists u T such that f (z) = uz, for every z D.
Proof. Since f has a zero at 0, it follows that the map f (z)/z has a removable
singularity there. (Write f as power series about the origin and factor.) Thus, there
exists g H(D) such that f (z) = zg(z). Now, fix z D\ {0}. Then for any r such
that |z| < r < 1 (such an r exists, as D is open), we have, by the Maximum Modulus
Principle, that

f (rei )

1
|f (z)|
i
= |g(z)| max g(re ) = max
,

|z|
r
r

27

since |f (z)| 1. Letting r 1 and multiplying through by |z|, gives |f (z)| |z|,
provided z D\ {0}. Since this inequality is evidently also true at z = 0, we get
(4.1.1). Now, (4.1.2) follows from the fact that f 0 (0) = g(0), by the product rule.
Finally, if equality holds in (4.1.1) for some z D0 or if equality holds in (4.1.2),
then application of Maximum Modulus Principle to g again shows that f (z)/z is a
constant of unit modulus.
(b) Since f is conformal automorphism of D, f 1 is as well (see Theorem 10.33 in
[RUD]). Thus, f and f 1 both satisfy the hypothesis of the Schwarz Lemma, and
therefore,

|f 0 (0)| , (f 1 )0 (0) 1.

(4.1.3)

Since z = (f f 1 )(z), for all z D, we get


1 = f 0 [f 1 (0)](f 1 )0 (0) = f 0 (0)(f 1 )0 (0),

(4.1.4)

by the Chain Rule. Combining (4.1.3) and (4.1.4), we must have |f 0 (0)| = 1, allowing
us to deduce from the second part of the Schwarz Lemma that f is a rotation.

Problem 4.2. Let a D and define fa (z) :=

az

, for z D.
1a
z

(a) Show that fa is an automorphism of D and is its own inverse.


(b) Show that for every conformal automorphism f of D there exists u T and
a D such that f = u fa .
(c) Verify that (b) implies that every conformal automorphism of D extends to

a homeomorphism of D.
(d) Show that the values a and u are uniquely determined by f .
Key terms: conformal mapping, Maximum Modulus Principle, Chain Rule, homeomorphism.

28

Solution. (a) If a = 0, then fa (z) = z, and so, (a) is immediate, in this case. So,
The computation
we may assume that a D\ {0}. Note that this forces a
1 C\D.
az
a 1
az
(fa fa )(z) =
az =
1a
1az

a(1
az)(az)
1
az
1
az
a(az)
1
az

a |a|2 z a + z
=
1a
z |a|2 + a
z

z |a|2 z
=
=z
1 |a|2
shows simultaneously that, on C\ {a1 }, fa acts as both left and right inverse to itself.
onto D.
We
Thus, fa is a bijection on C\ {
a1 }. It remains to show that fa maps D
will, in fact, prove a stronger result: fa maps T onto T and D onto D. To see this,
let u T. Then

au
|a u|
|a u|

=
|fa (u)| =
=
= 1.

1
1a
u
|u(
a u )|
|u| |a u|
It follows that fa (T) T, and so, T = (fa fa )(T) fa (T). These inclusions combine
to give fa (T) = T. Since is fa is nonconstant, we may deduce from the Maximum
Modulus Principle that |fa (z)| < sup {|fa (w)| : w D = T} = 1, for every z D.
Thus, fa (D) D. Applying fa to both sides reverses the inclusion, so that fa (D) = D.
(b) Put a := f 1 (0) D. Since f and fa are both conformal automorphisms of
D, it follows from the Chain Rule that f fa is also a conformal automorphism of
D. Moreover, (f fa )(0) = f [fa (0)] = f (a) = f [f 1 (0)] = 0; i.e., f fixes 0. Part (b)
of Problem 4.1 above yields u T such that (f fa )(z) = uz, for every z D. In
consequence, for every z D, we have
f (z) = f [(fa fa )(z)] = (f fa )[fa (z)] = ufa (z).
and therefore, their
(c) fa and multiplication by u are both homemorphisms of D,

composition is a homeomorphism of D.
(d) This amounts to proving that if u, v T and a, b D and
(4.2.1)

ufa (z) = vfb (z)

(z D),

then u = v and a = b. To verify that this is true, first observe that since (4.2.1) holds
ab
= 0. Since u 6= 0, we conclude that a b = 0, and therefore, a = b.
for z = b, u 1
ab

29

Thus, for every z D, we have ufa (z) = vfa (z), or equivalently, (u v)fa (z) = 0,
for every z D. Since fa is not the zero function, it must be that u v = 0, and so,
u = v.

Problem 4.3. Suppose the series

cn z n converges in D and the function f (z)

n=0

that it defines vanishes at k1 , for each k N. Prove that f 0.


Key terms: power series, Well-Ordering Principle in N0 .
Solution. Assume, with a view to reach a contradiction, that there exists n N0
such that cn 6= 0. By the Well-Ordering Principle, the set {n N0 : cn 6= 0} has a
P
n
least element, say N . Put g(z) :=
n=0 cN +n z . By the very definition of N , it must
be that cn = 0, for n = 0, 1, ..., N 1. In consequence,
f (z) =
From this and the fact that f

1
n

cn z n = z N g(z).

n=N

= 0, for every n N, we conclude that g

1
n

= 0,

for every n N. As g is represented by a convergent power series in D, it must be


that g H(D), whence g is continuous. Thus,


cN = g(0) = g lim n1 = lim g n1 = lim 0 = 0,
n

contradicting our choice of N . So, our initial assumption is false, from which we infer
that cn = 0, for every n N0 , and so, f 0.

Problem 4.4. Complete the following:


(a) State the Open-Mapping Theorem for holomorphic functions.
(b) State the Maximum Modulus Principle for holomorphic functions.
(c) Provide a short deduction of (b) from (a).
(d) Show that the image of any closed subset (of the plane) under a nonconstant
polynomial is closed.
(e) From (a) and (d) deduce the Fundamental Theorem of Algebra.

30

Key terms: Open-Mapping Theorem, Maximum Modulus Principle, Fundamental


Theorem of Algebra, Bolzano-Weierstrass Theorem.
Solution. (a) Here is the terse version found in [RUD, 214]:
Theorem 4.4.1 (The Open-Mapping Theorem). If is a region and f H(),
then f () is either a region or a point.
(b) Again, we provide Rudins version found on [RUD, 253]:
where
Theorem 4.4.2 (The Maximum Modulus Principle). If f H() C(),
is a bounded region, then the inequality
|f (z)| kf k := sup |f (w)|
w

holds for every z . Moreover, equality occurs at some point in if and only if f

is constant on .
(c) We may deduce (b) from (a) since no nonempty open subset of C contains an
element of maximum modulus. To see this, suppose is an open subset of C and
z0 . Since is open, we may find r > 0 such that D(z0 , r) . Put

z0 + z0 r z0 6= 0
2|z0 |
.
z1 :=

r
z0 = 0
2

Direct computation shows that |z1 z0 | =

r
,
2

and therefore, z1 D(z0 , r) .

Moreover, we also have that |z1 | > |z0 |. In summary, for any point z0 , we may
find another point z1 in that has larger modulus than z0 ; i.e., does not contain
a point of maximum modulus.
P
k
(d) Let p(z) := m
k=0 ak z be a nonconstant polynomial, with am 6= 0 (m > 0),
and let K be a closed subset of the plane. Let w p(K). Choose a sequence
{wn } p(K) such that wn w. As, each wn p(K), we may find zn K such that
p(zn ) = wn . Now, the triangle inequality gives

m1
m1

X
X

m
k
|ak | |z|k ,
ak z |p(z)| +
|am | |z| = p(z)

k=0

k=0

31

and therefore,
(4.4.1)

|z|m |am |

m1
X
k=0

|ak |
|z|mk

|p(z)|

(z 6= 0).

It follows easily from (4.4.1) that |p(z)| , as z , since am 6= 0. In consequence, the sequence {zn }, must be bounded, for otherwise there would be infinitely
many n N such that |wn | = |p(zn )| > |w| + 1, contradicting that |wn | |w|. Thus,
by the Bolzano-Weierstrass Theorem, we may find a subsequence {znl } that converges
to some element, say z, of K, since K is closed. Finally, from the continuity of p, we
get

w = lim wnl = lim p(znl ) = p lim znl = p(z),


l

and so, w p(K), proving p(K) is closed.


(e) Here is the form of the Fundamental Theorem of Algebra that we shall prove:
Theorem 4.4.3 (The Fundamental Theorem of Algebra). If p(z) is a polynomial
of positive degree m over C, then there are precisely m zeros of p in C, provided one
accounts for multiplicity.
Proof. Write p(z) :=

Pm
k=0

ak z k . Choose r > 0 such that |am | rm

Pm1
k=0

|ak | rk >

|a0 | = |p(0)|. Then, as in the proof of (d), the triangle inequality ensures that

p(rei ) > |p(0)|, for all . If p does not have any zeros, then the function f (z) :=

1/p(z) would be entire and satisfy |f (0)| > f (rei ), for all . This contradicts the
Maximum Modulus Principle. Thus p has at least one zero, say z0 , and so, we may
write p(z) = (z z0 )q(z), where q is some polynomial of degree m 1. By induction,
we may deduce that p has exactly m zeros, counting multiplicities.

Problem 4.5. Let X be a Lebesgue measurable subset of R with |X| = . Construct a function f such that f Lp (X), for every p 1, but f 6 L (X).
Key terms: Lebesgue measure, Lp -space, uniform continuity, Lebesgues Monotone
Convergence Theorem, Beppo-Levi Theorem, ratio test, essential supremum
Solution. We will use the following two general results:

32

Theorem 4.5.1. Let E be a Lebesgue measurable subset of R. Then the function


defined by fE (x) := |E (x, x)|, for x [0, ), is a uniformly continuous map
taking [0, ) onto [0, |E|). Furthermore, lim fE (x) = |E|.
x

Proof. Notice that we may write fE (x) =

R
R

1E 1(x,x) . Let x, y [0, ).

Without loss of generality, assume x y. Observe


Z
Z
Z

|fE (x) fE (y)| = 1E 1(x,x) 1E 1(y,y)


1E 1(x,x) 1(y,y)
R
R
R
Z

=
1E 1(y,x) + 1(x,y) = |E (y, x)| + |E (x, y)|
R

2(y x) = 2 |x y| .
The uniform continuity of f is now immediate.

Applying Lebesgues Monotone


Convergence
Theorem
to
the
sequence
1
1
E
(n,n)
Z

shows that lim fE (n) =


n

1E = |E|. As fE (0) = 0, we may deduce that fE maps


R

[0, ) onto [0, |E|) from the Intermediate-Value Theorem.

Proposition 4.5.2. Let {xn } [0, ) and X a Lebesgue measurable set of infinite measure. Then there exists a sequence {Xn } of pairwise disjoint Lebesgue measurable subsets of X such that |Xn | = xn , for each n N.
Proof. Let fX be as in Theorem 4.5.1. Since fX maps [0, ) onto [0, |X|) =
[0, ), we may find x [0, ) such that X1 = X (x, x) X has measure
x1 . Assume inductively that we have found pairwise disjoint measurable subsets
X1 , ..., Xn of X such that |Xk | = xk , for 1 k n. By additivity, we have that
T
P
P
P
|X| = |X ( nk=1 Xkc )| + nk=1 |Xk |. Since |X| = and nk=1 |Xk | = nk=1 xk < ,
T
it must be that |X ( nk=1 Xkc )| = . Mimicking the construction of X1 , we may
T
find Xn+1 X ( nk=1 Xkc ) X such that |Xn+1 | = xn+1 . That Xn+1 is disjoint from
each Xk , 1 k n is obvious. This completes the induction and yields the desired
sequence of sets.

33

Returning to Problem 4.5, by the results above, we may find a sequence {Xn } of
pairwise disjoint measurable subsets of X such that |Xn | = 2n , for every n N.

X
Put f :=
n1Xn . Since f is nonnegative and since the Xn are pairwise disjoint,
n=1
p

it follows that |f | = f =

np 1Xn . Thus, the Beppo-Levi Theorem and the ratio

n=1

test for series give


Z
Z X

X
X
X
p
p
p
p
|f | =
n 1Xn =
n 1Xn =
n |Xn | =
np 2n < ,
X

X n=1

n=1

n=1

n=1

whence f Lp (X).
To verify that f 6 L (X), set SM := {x X : |f (x)| > M }, for M 0. Since
we may always find n N so large so that M < n and since |f | = f , we have that
SM {x X : f (x) = n} = Xn .
By monotonicity, |SM | |Xn | = 2n > 0, and consequently, the set {M : |SM | = 0}
is empty. Hence, the essential supremum kf k = , and therefore, f 6 L (X).

Problem 4.6. Let (X, A , ) be a finite positive measure space. Suppose f


R
L (X) is such that kf k > 0. For n N, put n := X |f |n d = kf knn . Show that
(a) kf kn kf k , as n and
(b) n+1 /n kf k , as n .
Key terms: essential supremum, finite measure space, positive measure.
Solution. (a) f L (X) means that kf k < , where
kf k := inf {M : ({x : |f (x)| > M }) = 0} .
So, for every > 0, the set S := {x X : |f (x)| > kf k } is nonempty, and
moreover, (S ) > 0. Since |f | > kf k on S , we have the following
Z
Z
n
n
0 < (kf k ) (S ) =
(kf k ) d
|f |n d
S

|f |n d = kf knn ,

34

for every n N, provided < kf k . Taking the nth root gives


1

[(S )] n (kf k ) kf kn .
1

Since 0 < (S ) (X) < , we have that lim [(S )] n = 1, whence


n

kf k lim inf kf kn .
n

As this is true for arbitrary > 0, we get


(4.6.1)

kf k lim inf kf kn .
n

For the reverse inequality, we recall that |f | kf k a.e. Hence, |f |n kf kn a.e.,


for each n N. So, we obtain
Z
Z
n
n
kf kn =
|f | d
kf kn d = kf k (X).
X

Taking the nth root, gives


1

kf kn kf k [(X)] n .

(4.6.2)

Now, since 0 < kf k , we must necessarily have 0 < (X), and so, lim [(X)] n = 1.
n

By taking the limit superior in (4.6.2), we get lim sup kf kn kf k . This with (4.6.1)
n

proves that lim kf kn exists and is equal to kf k .


n
R
(b) Combining X |f |n d kf kn (X) < with (4.6.1), we see that n
(0, ), for every n N. As
Z
Z
Z
n+1
n
|f |
d =
|f | |f | d kf k
|f |n d,
X

we obtain n+1 kf k n , for every n, so that


(4.6.3)

lim sup
n

n+1
kf k .
n

Now, Holders Inequality yields


n Z
1
Z
n+1
n+1
Z
n
1
n
n n+1
n+1
n =
|f | d
(|f | ) n d
= (n+1 ) n+1 [(X)] n+1
1X d
X

n+1 [(X)]
kf kn+1

1
n+1

35

or equivalently,
(4.6.4)

kf kn+1
[(X)]

1
n+1

n+1
n

(n N).
1

Part (a) along with the fact that lim [(X)] n+1 = 1 (as (X) (0, )) yields
n

(4.6.5)

kf k = lim

kf kn+1
[(X)]

1
n+1

lim inf
n

n+1
,
n

by (4.6.4). Consideration of (4.6.5) along side (4.6.2) shows that lim n+1 /n exists,
n

and moreover, kf k = lim n+1 /n .


n

Problem 4.7. For p R, define hp :=

P
n=1

np 1In , where In :=

1
,1
n+1 n

. Prove

(a) hp L1 (R), provided p < 1.


(b) h1 L1weak (R)\L1 (R).
(c) hp 6 L1weak (R), for p > 1.
Key terms: Lp -space, Weak Lp , Beppo-Levi Theorem, p-test.
Solution. (a) Observe that

Z
Z X
X Z
X 1
1
p
p
p
(4.7.1)
hp =

n 1In =
n
1In =
n
n n+1
X
X np1
X 1
np
=
=

,
n(n + 1)
n+1
n2p
where the change of integration and summation is justified by the Beppo-Levi Theorem. The series on the right of (4.7.1) converges by the p-test for series, since p < 1.
(b) For R, put S := {x R : |h1 (x)| > }. Fix 0. Define N :=
max {n N : n }. Note that N , while N + 1 > .
It follows that SN S so that |S | |SN |. Now, from the definition of h1
`
we readily see that h1 (x) = |h1 (x)| > N if and only if x
n=N +1 In . Thus,
`
SN = n=N +1 In , and therefore,

X
1
1
1
1
1
1

lim
=
< .
|SN | =
=
n n+1
N + 1 n n + 1
N +1

n=N +1
In summary,
|S | |SN | < (1/) = 1.

36

As 0 is arbitrary, we may take the supremum over all such to conclude that
h1 L1weak (R).
To see that h1 6 L1 (R), we note that the computation involved in (4.7.1) is valid
R
P 1
. Since the series is not finite, neither is the integral.
for h1 . That is, h1 =
n+1
(c) First, let us set some notation. For x R, we define [x] := max {n Z : n x}.
Now, analogous to the above, let S := {x R : |hp (x)| > }. Fix p and and set
N := min {n N : np }. By definition, N p , while (N 1)p < . Now,
`
x SN if and only if hp (x) = |hp (x)| > N if and only if x
n=[N 1/n ]+1 In so that
`
SN = n=[N 1/n ]+1 In , and therefore, |SN | = [N 1/p1 ]+1 . Since N , SN S , and so,
|S | |SN | >

(N 1)p
1

[N p ] + 1

(N 1)p
1

[N p ] + 1

(N/2)p
1

2N p

N p p
= p+1 .
2

Now, since p > 1, N , as . Hence, we conclude that hp 6 L1weak (R).

Problem 4.8. Choose intervals Jn (0, 1) in such a way that U =

S
n

Jn is

dense in (0, 1) and yet the set K := (0, 1)\U has positive measure.
Key terms: Lebesgue measure
Solution. Let 0 < < 1 and put Q := Q (0, 1). Note that Q is dense in (0, 1),
yet has measure zero, since it is countable. By the definition of Lebesgue (outer)
P
measure, we may find open intervals In such that n In covers Q and n |In | < .
Put Jn := In (0, 1). Then each Jn is an interval (or empty),Jn (0, 1), for each
P
S
S
S
n, and moreover, U = n Jn n In . Hence, |U | = | n In | n |In | < . Since
(0, 1) has finite measure, we get
|K| = |(0, 1)\U | = |(0, 1)| |U | = 1 |U | > 1 > 0.

37

CHAPTER 5

Spring 2006
Problem 5.1. Let (X, A , ) be a finite measure space. Prove:
(a) For 1 r s , we have Ls (X) Lr (X).
(b) The result of (a) can fail if (X) = .
Key terms: finite measure space, Lp -space, Holders Inequality, Lebesgue Measure,
conjugate exponent.
Solution. (a) Clearly, we may assume that r < s, so that 1 < rs . Put p :=

s
r

and let

q be its conjugate exponent.


For f Ls (X), Holders inequality gives
1/q
Z
1/p Z
Z
Z
r p
r
r
q
(|f | )
1X d
|f | d =
|f | 1X d
X

1/p Z

|f |rp

1X d
X

1/q

= kf krs [(X)]1/q < .

Consequently, f Lr (X). As this is true for each f Ls (X), the conclusion follows.
(b) Let X := [1, ), A := Lebesgue measurable subsets of [1, ), and :=
Lebesgue measure. Then for f (x) := x1 , we have
Z
Z
1
2
|f | d =
dx = 1,
x2
X
1
while

|f | d =
X

1
dx = .
x

Hence, in this situation, we see that L (X) 6 L1 (X), even though 1 2.

Problem 5.2. Let A, B R be Lebesgue measurable and define h(x) := |(A x) B|.
R
Show that (a) h is a Lebesgue measurable function; and, (b) R h(x)dx = |A| |B|.

38

Key terms: measurable function, measurable set, convolution, Lebesgues Monotone


Convergence Theorem, Fubinis Theorem, translation invariance.
Solution. For n N, define An := A (n, n) and Bn := B (n, n). Notice that
|An | , |Bn | 2n < so that 1An , 1Bn L1 (R). Consequently, by Theorem 8.14 in
[RUD] (the Convolution Theorem), the function given by
Z
hn (x) :=
1An (x + y)1Bn (y)dy
R

is Lebesgue integrable. In particular, each hn is Lebesgue measurable.

Fix x R and observe that 1(An x)Bn n=1 is a monotone increasing sequence
of measurable functions that converge everywhere to 1(Ax)B , whence Lebesgues
Monotone Convergence Theorem yields
Z
Z
Z
1(Ax)B (y)dy = lim
1(An x)Bn (y)dy = lim
1An (x + y)1Bn (y)dy
n

lim hn (x).

On the other hand,


Z

h(x) = |(A x) B| =

1Ax (y)1B (y)dy =


R

1(Ax)B (y)dy,
R

so that h(x) = lim hn (x). Since x is arbitrary, we conclude that h is the pointwise
n

limit of a sequence of measurable functions, and so, h is measurable.


To show (b), we first notice that
1Ax (y) = 1A (x + y) = 1Ay (x)
holds for each x, y R. Combining this with Fubinis Theorem and the translationinvariance of Lebesgue measure gives

Z
Z Z
Z Z
h(x)dx =
1Ax (y)1B (y)dy dx =
1Ax (y)1B (y)dx dy
R

Z Z

1Ay (x)dx 1B (y)dy =


|A y| 1B (y)dy
R
Z
Z
=
|A| 1B (y)dy = |A| 1B (y)dy = |A| |B| .
R

39

Problem 5.3. Let A be a -algebra on a set X and assume : A [0, ] has


the following properties:
(i) If A1 , A2 F with A1 A2 = , then (A1 A2 ) = (A1 ) + (A2 ).
(ii) If {An }
n=1 is a sequence in A such that An+1 An , for all n N, and

\
An = , then lim (An ) = 0.
n

n=1

Prove that is a positive measure on X.


Key terms: -algebra, positive measure, finitely additive, countably additive.
Solution. Since A is a -algebra, we have A . For n N, let An = . The
sequence {An } satisfies the hypothesis of (ii), whence
() = lim () = lim (An ) = 0.
n

So, is not identically infinite.


It remains to be shown that is countably additive. Now, condition (i) combined
with induction readily shows that is finitely additive; i.e.,
k
!
k
[
X

An =
(An ),
n=1

n=1

whenever A1 , ..., Ak A are pairwise disjoint.


To see that is, in fact, countably additive, let {Bm }
m=1 be a sequence of mea
[
surable pairwise disjoint sets. For n N, put An :=
Bm . Clearly, An+1 An , for
m=n

each n. Now, suppose, in order to reach a contradiction, that there exists x


Then x A1 =

An .

n=1

Bm . Since the sets Bm are pairwise disjoint, there is a unique

m=1

n(x) N such that x Bn(x) . Hence, x 6

Bm = An(x)+1 . This contradicts

m=n(x)+1

that x

\
n=1

An . Thus,

An = , and therefore, {An } has the requisite properties

n=1

to invoke (ii). In consequence, lim (An ) = 0. Now, notice that as the Bm are
n

40

pairwise disjoint, the sets

!
Bm

Bm and An are disjoint, provided n 2. So, for each

m=1

n 2, we have

n1
[

" n1
[

m=1

!
Bm

#
An =

n1
[

m=1

!
Bm

+ (An ) =

m=1

n1
X

(Bm ) + (An ).

m=1

Taking the limit as n completes the proof.

Problem 5.4. ZLet h be a bounded Lebesgue measurable function on R having the


property that lim
h(nx)dx = 0, for every Lebesgue measurable subset E of finite
n E
Z
1
measure. Show that for every f L (R), we have lim
f (x)h(nx)dx = 0.
n

Key terms: Lebesgue measure, Lp -space, abstract integration, simple function.


Solution. Let s be a complex, measurable, simple function on R that vanishes
P
outside a set of finite measure. Write s = m
k=1 k 1Ak , where k C\ {0} and Ak
are Lebesgue measurable and pairwise disjoint. Note that since s vanishes outside a
set of finite measure, each Ak has finite measure. Thus, for every such simple function,
we have
Z "X
m

Z
lim

(5.4.1)

s(x)h(nx)dx =
R

lim

n
m
X

k=1

#
k 1Ak (x) h(nx)dx

k lim

k=1

1Ak (x)h(nx)dx

= 0.

Now, let f L1 (R) and let M 0 be a finite bound for h. Fix > 0. By Theorem
3.13 in [RUD], we may find a complex, measurable, simple function s that vanishes
outside a set of finite measure such that kf sk1 < /M . Now, for each n N, we

41

have
Z

f (x)h(nx)dx = [f (x) s(x)]h(nx)dx + s(x)h(nx)dx

R
R
R
Z

(5.4.2)
|f (x) s(x)| |h(nx)| dx + s(x)h(nx)dx
R
R
Z

M kf sk1 + s(x)h(nx)dx
R

< + s(x)h(nx)dx .
R

Letting n in (5.4.2) and appealing to (5.4.1) gives

lim
n f (x)h(nx)dx < ,
R

by the continuity of the absolute value function. The proof is completed by letting
0.

Problem 5.5. Let L : C be a continuous function having the property that


eL(z) = z, for all z . Prove that
(a) L H().
(b) There is no continuous map on D0 that acts as a right inverse for the exponential map on D0 .
Key terms: exponential map, complex differentiability, index of a closed curve (with
respect to a point), Fundamental Theorem of Calculus, complex logarithm.
Solution. (a) Note that since ez is never zero, we may deduce that 0 6 . Now,
fix z and h C\ {0}. Notice that L must be injective, for if L(a) = L(b), then
a = eL(a) = eL(b) = b. Thus,

L(z+h)L(z)
h

L(z + h) L(z)
=
h

6= 0, and therefore,

(z + h) z
L(z + h) L(z)

eL(z)+H(h) eL(z)
H(h)

42

eL(z+h) eL(z)
=
L(z + h) L(z)

1
,

where H(h) := L(z + h) L(z). Since L is assumed continuous, we have H(h) 0,


as h 0. Combining this with the continuity of the map w 7 w1 on C\ {0} and
the fact that ez is its own derivative yields
L(z)+H(h)
1
1
eL(z)+H(h) eL(z)
L(z + h) L(z)
e
eL(z)
lim
= lim
= lim
h0
h0
h0
h
H(h)
H(h)
L(z) 1
= e
= z 1 .
This shows that L is differentiable on , and moreover, L0 (z) = z 1 .
(b) We will actually show that under the assumptions on L, D0 (0, r) 6 , for all
r > 0. To verify this, suppose contrarily that, for some 0 < r < , L is a continuous
right inverse for the exponential map in D(0, r), that is, eL(z) = z, for all 0 < |z| < r.
Put := 2r . Let : [0, 1] C be the closed curve given by (t) := e2it . By part
(a) L is differentiable and L0 (z) = 1/z. Therefore,
Z
Z
0
L (z)dz = z 1 dz = Ind (0) = 2i.

On the other hand, by the Fundamental Theorem of Calculus,

L0 (z)dz = 0, since

is closed. This contradiction completes the proof.

Problem 5.6. Let f and g be entire functions such that |f | |g|. Prove
(a) If z0 is a zero of g having multiplicity m, then z0 is a zero of f having
multiplicity at least m.
(b) f is a constant multiple of g.
Key terms: entire function, Liouvilles Theorem, zero-set, removable singularity.
Solution. It is enough to prove (b). To this end, note that it is clear that f vanishes
whenever g does. Thus, we may assume that g is not identically equal to zero. As
such, the set of zeros of g, Z(g), contains no limit point in C. In consequence, for
each (fixed) zero z Z(g), we may find rz > 0 such that D0 (z, rz ) Z(g) = . Hence,
the function

f
g

is holomorphic and bounded (by 1) on the punctured disk D0 (z, rz ),

and therefore, has a removable singularity at z. (See Theorem 10.20 in [RUD].) Let

43

hz be an extension of
since this is true for

f
g

f
g

to a holomorphic function on D(z, rz ). Note that |hz | 1,

on D0 (z, rz ) and since hz is continuous (it is holomorphic) on

D(z, rz ).

f (z)

Now, define h : C C by h(z) =

g(z)

z 6 Z(g)

. Clearly, h is differentiable

hz (z) z Z(g)
at every z in the open set C\Z(g). Furthermore, for z Z(g), h and hz agree in the
neighborhood D(z, rz ) of z, where hz is holomorphic, and so, h is also differentiable
at z. We conclude that h is an entire function. Moreover, the conditions on f and
g and the fact that |hz | 1, for all z Z(g), ensure that |h| 1. By Liouvilles
Theorem, we deduce that h is some constant c. The very definition of h thus gives
that f (z) = cg(z), for all z 6 Z(g). Yet |f | |g| implies that f (z) = 0 = c 0 = cg(z),
for all z Z(g). So, we actually have f (z) = cg(z), for all z C.

Problem 5.7. Verify that

Z
2
dx
(a)
=
.
4
1+x
2
Z
2
(b)
ei exp(ei )d = 2.
0

Key terms: contour integration, Residue Theorem, pole, Laurent series.


Solution. (a) Put f (z) :=

1
z 4 +1

h
i
(2k+1)
and note that f has simple poles at zk = exp i 4
,

k = 0, ..., 3. For R > 1, let R : [0, ] C be the path t 7 Reit . Among the four
simple poles, only z0 and z1 lie inside the region bounded by R and the x-axis. Hence,
Z R
Z
3
X
Res(f, zk ),
f (x)dx +
f (z)dz = 2i
R

k=0

or equivalently,
Z

(5.7.1)
R

1
dx =
x4 + 1

Now, observe that


Z
Z

f (z)dz =

Z
f (z)dz + 2i [Res(f, z0 ) + Res(f, z1 )] .
R

1
R
R
it

Rie
dt
dt
=
.

4
(Reit )4 + 1
R4 1
0 R 1

44

Clearly, the above inequality implies that R f (z)dz 0, as R . Applying


this fact to (5.7.1) we may conclude that
Z
1
(5.7.2)
dx = 2i [Res(f, z0 ) + Res(f, z1 )] .
4
x + 1
At this stage, it is appropriate to calculate Res(f, z0 ) and Res(f, z1 ). The procedure is the same for both:
Res(f, zk ) = lim

zzk

z zk
=
z4 + 1

1
=
z 4 zk4
lim
zzk z zk

d 4
z z=z
dz
k

1
.
4zk3

So,

+ exp i 9
exp i 3
2i sin (/4)
i
4
4
Res(f, z0 ) + Res(f, z1 ) =
=
= .
4
4
2 2
Substitution of this into (5.7.2) gives the desired result.
(b) Let be the closed curve given by 7 ei , for [0, 2]. Then
Z 2
Z z
e
i
i
(5.7.3)
e exp(e )d = i
dz.
2
0
z
By the Residue Theorem, one has
z
Z z
e
e
dz
=
2iRes
, 0 = 2i,
2
z2
z
since

ez
z2

has Laurent expansion

1
z2

+ z1 + 12 + 3!z +... about the point z = 0. Substituting

this into equation (5.7.3) completes the solution.

Problem 5.8. Let S := {z C : 0 < Re z < 2} and for each z S let lz (t) :=
tz + 1 t, for 0 t 1. Without assuming that a primitive of f exists, show that
R
F (z) := lz f defines a holomorphic function in S such that F 0 = f . Explain why the
conclusion fails whenever f is not holomorphic.
Key terms: Cauchys Theorem in a Convex Set, complex differentiability.
Solution. This is problem is a specific example of the following more general result,
whose proof follows the statement:

45

Theorem 5.8.1 (Cauchys Theorem in a Convex Set). Let f be a function defined


on a convex open subset of C. Show that f H() if and only if there exists
F H() such that F 0 = f .
Proof. () Fix a . Since is convex,
Z it contains the line segment [a, z], for
every z . Define F : C C by F (z) :=
f ()d. Now, fix z0 . For any
z , Cauchys Theorem for a triangle gives
Z
Z
Z
f ()d =
f ()d
[z,z0 ]

[z0 ,a]

[a,z]

[a,z]

Z
f ()d =

f ()d F (z)
[a,z0 ]

= F (z0 ) F (z).
Hence, for z \ {z0 }, we have

F (z0 ) F (z)

f (z0 ) =

z0 z
(5.8.1)

Z
Z
1

f ()d
f (z0 )d
z0 z
z0 z [z,z0 ]
[z,z0 ]
Z
1

|f () f (z0 )| d || .
|z0 z| [z,z0 ]

Let > 0 be given. Using the continuity of f , choose > 0 so small so that
| z0 | < implies and |f () f (z0 )| < . Now, if 0 < |z z0 | < , then
| z0 | < , for each [z, z0 ]. Thus, from (5.8.1) we may deduce that

Z
F (z0 ) F (z)

f (z0 ) <
d || = ,

z0 z
|z0 z| [z,z0 ]
for all z with 0 < |z z0 | < . This proves that F is differentiable at z0 , and
moreover, F 0 (z0 ) = f (z0 ). As z0 is arbitrary, the proof of this implication is
complete.
() As holomorphic functions have complex derivatives of all orders, F H()
with F 0 = f implies that f H().

46

Index

additivity

enssential supremum, 32, 34

countable, 40

entire function, 8, 11, 21, 43

finite, 40

exponential map, 42

almost everywhere (a.e.), 7, 12


Fubinis Theorem, 1, 15, 39
Beppo-Levi Theorem, 7, 9, 32, 36

Fundamental Theorem of Algebra, 31, 32

Bolzano-Weierstrass Theorem, 20, 31

Fundamental Theorem of Calculus, 42

Casorati-Weierstrass Theorem, 11

Holders Inequality, 38

Cauchys Formula, 16

harmonic function, 21

Cauchys Theorem

homeomorphism, 28

in a Convex Set, 45, 46


Chain Rule, 27, 28

index of a closed curve, 2, 42

complex logarithm, 42

integration

conformal mapping, 27, 28

abstract, 41

conjugate exponent, 38

contour, 3, 18, 44

convergence
in measure, 23

Laurent series, 44

p-norm, 7

Lebesgue

pointwise, 7, 23

Dominated Convergence Theorem, 1, 4

uniform, 2, 22

measure, 46, 12, 19, 32, 37, 38, 41

convolution, 39

Monotone Convergence Theorem, 9, 20, 32,

counting measure, 6

39
point, 12

differentiability

Liouvilles Theorem, 8, 21, 43


Lp -space, 6, 16, 19, 20, 26, 32, 36, 38, 41

complex, 4, 42, 45

47

uniform continuity, 32

Maximum Modulus Principle, 12, 20, 27, 28,

Uniqueness Theorem for Holomorphic

31

Functions, 22

measurable
function, 4, 39

variation

set, 5, 39

bounded, 10

measure

total, 10

positive, 34, 40
measure space

weak Lp , 36

complete, 5

Well-Ordering Principle (in N), 30

finite, 26, 34, 38


metric space

zero-set, 20, 43

complete, 7
Moreras Theorem, 1
negligible set, 5
Open-Mapping Theorem, 31
p-test, 36
polar coordinates, 15, 16
pole, 3, 11, 18, 44
power series, 22, 30
ratio test, 32
residue, 3, 11, 18
Residue Theorem, 3, 18, 44
rotation, 27
Schwarz Lemma, 27
-algebra, 40
simple function, 41
singularity
removable, 11, 27, 43
translation invariance, 39

48

Bibliography
[CAB] R. V. Churchill, J. W. Brown,
Complex Variables and Applications. (Seventh Edition)
McGraw-Hill (2003)
[PAM] M. H. Protter, C. B. Morrey,
A First Course in Real Analysis. (Second Edition)
Springer-Verlag New York, Inc. (1991)
[ROY] H. L. Royden,
Real Analysis. (Third Edition)
Prentice Hall, Inc. (1988)
[RUD] W. Rudin,
Real and Complex Analysis. (Third Edition)
McGraw-Hill. (1987)

49

You might also like